Đến nội dung

Hoang Nhat Tuan nội dung

Có 1000 mục bởi Hoang Nhat Tuan (Tìm giới hạn từ 02-06-2020)



Sắp theo                Sắp xếp  

#559505 $ \sqrt{\dfrac{a}{b+c}}+\sq...

Đã gửi bởi Hoang Nhat Tuan on 15-05-2015 - 12:33 trong Bất đẳng thức và cực trị

1. Cho $a,b,c>0$ . Chứng minh: $ \sqrt{\dfrac{a}{b+c}}+\sqrt{\dfrac{b}{a+c}}+\sqrt{\dfrac{c}{a+b}} > 2 $

 

2. Cho $x\ge 1, y \ge 1 $. Chứng minh: $ x\sqrt{y-1}+y\sqrt{x-1} \le xy $

Câu 1: $\sum \sqrt{\frac{a}{b+c}}=\sum \frac{a}{\sqrt{a(b+c)}}\geq \frac{2a}{a+b+c}=2$

Dấu "=" không xảy ra  :biggrin:




#571854 $ \sum \frac{1}{a+2b+3c}<\frac...

Đã gửi bởi Hoang Nhat Tuan on 12-07-2015 - 21:31 trong Bất đẳng thức và cực trị

Từ giả thết rút ra được: $\sum \frac{1}{a}=1$

Áp dụng BĐT Cauchy-schwarz:

$\sum (\frac{1}{a}+\frac{2}{b}+\frac{3}{c})\geq \sum \frac{36}{a+2b+3c}$

Do đó:$\sum \frac{1}{a+2b+3c}\leq \frac{1}{6}<\frac{3}{16}$




#571858 $ \sum \frac{1}{a+2b+3c}<\frac...

Đã gửi bởi Hoang Nhat Tuan on 12-07-2015 - 21:35 trong Bất đẳng thức và cực trị

Bài 2:$\frac{6}{2(ab+bc+ca)}+\frac{2}{a^2+b^2+c^2}\geq \frac{2(\sqrt{3}+1)^2}{1}>14$

Áp dụng BĐT Cauchy-schwarz




#571283 $ \sum \frac{a^{3}}{(a+b)^{3...

Đã gửi bởi Hoang Nhat Tuan on 11-07-2015 - 00:22 trong Bất đẳng thức và cực trị

Cho $a,b,c$ là những số dương . Chứng minh rằng 

$\frac{a^{3}}{(a+b)^{3}}+\frac{b^{3}}{(b+c)^{3}}+\frac{c^{3}}{(c+a)^{3}}\geq \frac{3}{8}$

Ta có:$3(\sum \frac{a^3}{(a+b)^3})^2\geq (\sum \frac{a^2}{(a+b)^2})^3$ (BĐT Holder)

Cần chứng minh:$\sum \frac{a^2}{(a+b)^2}\geq \frac{3}{4}$

Giờ đặt:$x=\frac{b}{a};y=\frac{c}{b};z=\frac{a}{c}$ thì $xyz=1$

BĐT viết lại thành:

$A=\sum \frac{1}{(1+x)^2}\geq \frac{3}{4}$

Lại có:$\frac{1}{(1+x)^2}+\frac{1}{(1+y)^2}\geq \frac{1}{1+xy}$ (BĐT rất quen thuộc, chứng minh bằng BĐTĐ)

Do đó:$A\geq \frac{1}{1+xy}+\frac{1}{(1+z)^2}=\frac{z}{z+1}+\frac{1}{(1+z)^2}=\frac{(z-1)^2}{4(z+1)^2}+\frac{3}{4}\geq \frac{3}{4}$

BĐT được chứng minh :D




#578568 $ 2x+y+xy \geq 6$. Min $x^3+y^3=?$

Đã gửi bởi Hoang Nhat Tuan on 04-08-2015 - 20:30 trong Bất đẳng thức và cực trị

Cho $x,y \geq 0$ thỏa mãn $ 2x+y+xy \geq 6$. Tìm Min $x^3+y^3$

Từ giả thiết suy ra: $y\geq \frac{6-2x}{(x+1)}$

Do đó: $x^3+y^3\geq \frac{(6-2x)^3}{(x+1)^3}+x^3$

$f'(x)=3(x^2+\frac{8(x-3)^3}{(x+1)^4}-\frac{8(x-3)^2}{(x+1)^3})$

$f'(x)=0<=>x^2(x+1)^4+8(x-3)^3-8(x-3)^2(x+1)=0$

Đến đây ra được 2 nghiệm, loại nghiệm âm: http://www.wolframal...8(x-3)^2(x+1)=0

Tiếp theo lập bảng biến thiên rồi......

Spoiler




#570588 $(\sqrt{x^{2}+5}-4)(3x^{2}+1)=3+4x^...

Đã gửi bởi Hoang Nhat Tuan on 08-07-2015 - 19:56 trong Phương trình, hệ phương trình và bất phương trình

$(\sqrt{x^{2}+5}-4)(3x^{2}+1)=3-4x^{2}$

Ta có:$(\sqrt{x^2+5}-3)(3x^2+1)=4-x^2$

$<=>\frac{(x^2-4)(3x^2+1)}{\sqrt{x^2+5}+3}+x^2-4=0<=>(x^2-4)(\frac{3x^2+1}{\sqrt{x^2+5}+3}+1)=0$

Do đó $x=2$ hoặc $x=-2$




#565989 $(1-a)(1-b)(1-c)+\frac{a}{b+c+1}+\frac...

Đã gửi bởi Hoang Nhat Tuan on 15-06-2015 - 17:41 trong Bất đẳng thức và cực trị

bạn có thẻ viết rõ ra không! :icon6:

Vì giả sử $1\geq c\geq b\geq a\geq 0$ nên $b+c+1\geq a+b+1=>\frac{a}{b+c+1}\leq \frac{a}{a+b+1}$

Tương tự thì: $\frac{b}{a+c+1}\leq \frac{b}{a+b+1}$

Do đó: $\frac{a}{b+c+1}+\frac{b}{a+c+1}+\frac{c}{a+b+1}\leq \frac{a+b+c}{a+b+1}$ :D




#565983 $(1-a)(1-b)(1-c)+\frac{a}{b+c+1}+\frac...

Đã gửi bởi Hoang Nhat Tuan on 15-06-2015 - 17:17 trong Bất đẳng thức và cực trị

Cho $a,b,c \epsilon \left [ 0;1 \right ]$. CMR:

$(1-a)(1-b)(1-c)+\frac{a}{b+c+1}+\frac{b}{c+a+1}+\frac{c}{a+b+1}\leq 1$

Giả sử $1 \geq c \geq b \geq a \geq 0$

Sử dụng AM-GM:

$(1-a)(1-b)(1+a+b)\leq 1$

$<=> (1-a)(1-b)(1-c)\leq \frac{1-c}{a+b+1}$

Lại có:$\sum \frac{a}{b+c+1}\leq \frac{a}{a+b+1}+\frac{b}{a+b+1}+\frac{c}{a+b+1}\leq \frac{a+b+c}{a+b+1}$

Kết hợp với BĐT trên => ĐPCM




#568577 $(2+a^2)(2+b^2)(2+c^2) \geq 9(ab+bc+ca)$

Đã gửi bởi Hoang Nhat Tuan on 27-06-2015 - 23:47 trong Bất đẳng thức và cực trị

Cho a,b,c là các số thực. Chứng minh rằng:

 a. $a^2+b^2+c^2+2abc+1 \geq 2(ab+bc+ca)$

 b. $(2+a^2)(2+b^2)(2+c^2) \geq 9(ab+bc+ca)$

Cả 2 câu này chú ý sử dụng nguyên lý Dirichlet là được:

a) Áp dụng Dirichlet giả sử $(a-1)(b-1) \geq 0$ => $c(a-1)(b-1) \geq 0$

Do đó: $a^2+b^2+c^2+2abc+1-2(ab+bc+ca)=(a-b)^2+(c-1)^2+2c(a-1)(b-1) \geq 0$

b) Cũng tương tự như trên giả sử $(a^2-1)(b^2-1) \geq 0$

Từ đó chứng minh được: $(2+a^2)(2+b^2)\geq 3(a^2+b^2+1)$

Thay vào sử dụng Cauchy-schwarz là được :D




#572749 $(a+b^{2})(b+c^{^{2}})(c+a^{2})\leq 13+abc$

Đã gửi bởi Hoang Nhat Tuan on 15-07-2015 - 16:17 trong Bất đẳng thức và cực trị

Em gái Nhật Tuấn vào giúp với nào

với a, b, c là các số thực dương thỏa mãn a+b+c=3

$(a+b^{2})(b+c^{^{2}})(c+a^{2})\leq 13+abc$

Ta có một kết quả mạnh hơn ở đây:http://diendantoanho...b2bc2ca2leq-13/

Hình như không có dấu bằng :luoi:

Spoiler




#609733 $(a^{2}+b^{2}+abc)(b^{2}+c^{2}+a...

Đã gửi bởi Hoang Nhat Tuan on 18-01-2016 - 22:30 trong Bất đẳng thức và cực trị

$i)$ Cho các số dương $a;b;c$ thỏa mãn $a^{2}+b^{2}+c^{2} \geq 3$
Chứng minh rằng $(a^{2}+b^{2}+abc)(b^{2}+c^{2}+abc)(c^{2}+a^{2}+abc) \geq 3abc(a+b+c)^{2}$
$ii)$ Cho $a;b;c>0$ và $a+b+c=3$. Chứng minh rằng

$\frac{a}{3a^{2}+abc+27}+\frac{b}{3b^{2}+abc+27}+\frac{c}{3c^{2}+abc+27} \leq \frac{3}{31}$

$iii)$ Cho các số dương $a;b;c$ thỏa mãn $a+b+c=3$
Chứng minh rằng $\sqrt{\frac{a+b}{c+ab}}+\sqrt{\frac{b+c}{a+bc}}+\sqrt{\frac{c+a}{b+ca}} \geq 3$

Câu $iii)$

Áp dụng BĐT AM-GM thì cần chứng minh: $(a+b)(b+c)(c+a)\geq (c+ab)(a+bc)(b+ca)$

Ta có: $(c+ab)(a+bc)\leq \frac{[c+a+b(c+a)]^2}{4}=\frac{(c+a)^2(b+1)^2}{4}$

Tương tự 2 BĐT kia rồi nhân lại AM-GM phát nữa là ra.

P/s: Thầy có ra bài iii) đâu nhỉ :D

Câu $ii)$ còn một khoảng "rất nhỏ" giữa $1$ với $1,05$ mà không ra, ức chế thật -_-




#609832 $(a^{2}+b^{2}+abc)(b^{2}+c^{2}+a...

Đã gửi bởi Hoang Nhat Tuan on 19-01-2016 - 18:30 trong Bất đẳng thức và cực trị

 Câu iii) UCT cho lẹ :D

 

 Ta có : BĐT$\Leftrightarrow \sum \left (\dfrac{a}{27}-\dfrac{a}{3a^2+abc+27}\right )\geq \dfrac{12}{31}$

                    $\Leftrightarrow \sum \dfrac{3a^3+a^2bc}{3a^2+abc+27}\geq \dfrac{12}{31}$

 Áp dụng BĐT Cauchy-Schwarz ta có :

$$\sum \dfrac{3a^3}{3a^2+a^2bc+27}\geq \dfrac{3(a^2+b^2+c^2)^2}{3(a^3+b^3+c^3)+3abc+81}$$

$$abc\sum \dfrac{a}{3a^2+abc+27}\geq \dfrac{9abc}{3(a^3+b^3+c^3)+3abc+81}$$

 Nên ta chỉ cần chứng minh $\dfrac{3(a^2+b^2+c^2)^2+9abc}{3(a^3+b^3+c^3)+3abc+81}\geq \dfrac{12}{31}$

Hay $93(a^2+b^2+c^2)^2+279abc\geq 36(a^3+b^3+c^3)+36abc+972$

       $\Leftrightarrow 93(a^2+b^2+c^2)^2+243abc\geq 36(a^3+b^3+c^3)+972$

 Đặt $p=a+b+c=3;q=ab+bc+ca;r=abc$ thì ta cần chứng minh

       $93(9-2q)^2+243r\geq 36(27-9q+3r)+972$

       $\Leftrightarrow 124q^2-1008q+1863+45r\geq 0$

 Áp dụng BĐT Schur ta có $r\geq \dfrac{4q-9}{3}$ nên ta chỉ cần chứng minh

       $124q^2-1008q+1863+15(4q-9)\geq 0\Leftrightarrow (q-3)(31q-144)\geq 0$

 Luôn đúng vì $q\leq 3$

 Vậy BĐT được chứng minh xong. Dấu "=" xảy ra khi $a=b=c=1$

iii) UCT không được, tính đạo hàm cấp 2 ra $\leq 0$ nhé!




#570850 $(a^2+b^2+c^2)\left ( \frac{1}{a^2}+\...

Đã gửi bởi Hoang Nhat Tuan on 09-07-2015 - 22:07 trong Bất đẳng thức - Cực trị

Không biết có giống cách giải trong Topic kia không ?

Dễ thấy nếu $a,b,c$ đều âm thì giả thiết vô lí.

Giả sử có $2$ số $b,c<0$ và $a>0$. Khi đó đặt $b=-x, c=-y$

Ta có $10=[a-(x+y)][\frac{1}{a}-(\frac{1}{x}+\frac{1}{y})]\leqslant [a-(x+y)](\frac{1}{a}-\frac{4}{x+y})$

$\Rightarrow 5\leqslant -(\frac{x+y}{a}+\frac{4a}{x+y})$, vô lí.

Giả sử $b,c>0$ $a$ tùy ý

Áp dụng AM-GM ta có 

        $10=(a+b+c)(\frac{1}{a}+\frac{1}{b}+\frac{1}{c})\geqslant (a+b+c)(\frac{1}{a}+\frac{4}{b+c})$

$\Rightarrow 5\geqslant \frac{b+c}{a}+\frac{4a}{b+c}\Rightarrow \frac{b+c}{a} \in [1;4]$

Khi đó ta có 

  $P\geqslant [a^2+\frac{(b+c)^2}{2}][\frac{1}{a^2}+\frac{8}{(b+c)^2}]\geqslant \frac{27}{2}$

Câu này nằm ở đây:http://diendantoanho...các-nước/page-9




#565789 (a+b) (b+c) (c+a) $\geq$ (ab+c) (bc+a) (ca+b)

Đã gửi bởi Hoang Nhat Tuan on 14-06-2015 - 22:56 trong Bất đẳng thức và cực trị

Cho a,b,c là các số dương . Chứng minh rằng : $\frac{1}{a}+\frac{1}{b}+\frac{1}{c}\geq \frac{2a+b}{a\left ( a+2b \right )}+\frac{2b+c}{b\left ( b+2c \right )}+\frac{2c+a}{c\left ( c+2a \right )}$

Cho a,b,c > 0 và a+b+c=3 CMR: 

(a+b) (b+c) (c+a) $\geq$ (ab+c) (bc+a) (ca+b)

BĐT tương đương với: $2\sum \frac{1}{a}\geq \sum \frac{3a+a+2b}{a(a+2b)}=\sum (\frac{3}{a+2b}+\frac{1}{a})$

$=\sum \frac{3}{a+2b}+\sum \frac{1}{a}$

Lại có: $\sum \frac{3}{a+2b}\leq \sum \frac{1}{a}$ (dùng BĐT C-S theo chiều ngược)

=> ĐPCM




#572468 [CHUYÊN ĐỀ] CHỨNG MINH BẤT ĐẲNG THỨC

Đã gửi bởi Hoang Nhat Tuan on 14-07-2015 - 18:14 trong Bất đẳng thức và cực trị

Bài 31:Ta sẽ đặt: $0,999...9=a$

Ta sẽ chứng minh: $a<\sqrt{a}<1$

Thật vậy: Vì a<1 nên $a(a-1)<0$. Do đó $a^2<a$

Từ $a^2<a<1$ suy ra: $a<\sqrt{a}<1$

Do đó 20 chữ số đầu cần tìm là các chữ số 9 




#582200 [CHUYÊN ĐỀ] CHỨNG MINH BẤT ĐẲNG THỨC

Đã gửi bởi Hoang Nhat Tuan on 16-08-2015 - 01:09 trong Bất đẳng thức và cực trị

62.Phương trình bậc hai $x^{2}+mx+n=0 (n\leq m-1)$ có hai nghiệm là a và b. CMR:$a^{2}+b^{2}\geq 1$

P/s: Bài này chẳng biết là BĐT hay phương trình nữa (thấy nó trong tập BĐT của chị mình nên cho vào BĐT) :wacko:  :wacko: .Bài này trích Đề thi chọn HSG toán 9 toàn quốc bảng B, 1994- 1995.

Ta có: $\Delta =m^2-4n\geq 0=>m^2\geq 4n$

Sử dụng Vi-et thì: $a+b=-m;ab=n$

Khi đó: $a^2+b^2=m^2-2n\geq m^2-2(m-1)=m^2-2m+2\geq 1<=>(m-1)^2\geq 0$

BĐT được chứng minh

Spoiler




#572287 [CHUYÊN ĐỀ] CHỨNG MINH BẤT ĐẲNG THỨC

Đã gửi bởi Hoang Nhat Tuan on 14-07-2015 - 08:58 trong Bất đẳng thức và cực trị

 Đặt $\sqrt[3]{a}=x; \sqrt[3]{b}=y;\sqrt[3]{c}=z$

BĐT cần chứng minh trở thành: $x^3+y^3+z^3\geq 3xyz\Leftrightarrow (x+y)^3+z^3-3x^2y-3xy^2-3xyz\geq 0\Leftrightarrow (x+y+z)(x^2+2xy+y^2-xz-yz+z^2)-3xy(x+y+z)\geq 0\Leftrightarrow (x+y+z)(x^2+y^2+z^2-xy-yz-xz)\geq 0\Leftrightarrow (x+y+z)\left [ (x-y)^2+(y-z)^2+(z-x)^2 \right ]\geq 0$

(Luôn đúng với $x,y,z$ không âm)

 

P/s: Mình nghĩ đề bài bài 22 sai, hình như giả thiết phải là $\sum \frac{1}{1+a}\geq 3$

Mình thấy giả thiết đúng rồi đó, cách giải như thế này:

Ta sẽ đặt:

$x=\frac{a}{1+a};y=\frac{b}{1+b};z=\frac{c}{1+c};t=\frac{d}{1+d}$

Khi đó rút a theo x, b theo y, c theo z, d theo t, ta được bài toán mới:

Cho $x+y+z+t\leq 1$ Chứng minh rằng:

$\frac{xyzt}{(1-x)(1-y)(1-z)(1-t)}\leq \frac{1}{81}$

$<=>81xyzt\leq (1-x)(1-y)(1-z)(1-t)$

Lại có:$x+y+z+t\leq 1=>\prod (1-x)\geq \prod (y+z+t)$

Đến đây áp dụng AM-GM cho vế phải suy ra điều phải chứng minh

Spoiler




#572463 [CHUYÊN ĐỀ] CHỨNG MINH BẤT ĐẲNG THỨC

Đã gửi bởi Hoang Nhat Tuan on 14-07-2015 - 18:08 trong Bất đẳng thức và cực trị

đặt $P=\sqrt{2+\sqrt{2+...+\sqrt{2+\sqrt{2}}}}$

ta có $P< \sqrt{2+\sqrt{2+...+\sqrt{2+\sqrt{2+2}}}}=2$

=>Đpcm

Cần nói rõ hơn một tí:

Ta có:$\sqrt{2+\sqrt{2+\sqrt{2+...+\sqrt{2+\sqrt{2}}}}}=A$

Khi đó: $A^2=2+\sqrt{2+\sqrt{2+\sqrt{2+...+\sqrt{2+\sqrt{2}}}}}=2+A$

Do đó:$A^2-A-2=0<=>(A-2)(A+1)=0$

=> $A=2$

Từ đó suy ra ĐPCM

Bài 35: Áp dụng AM-GM ta cần chứng minh:

$\sum \frac{1}{2a\sqrt{bc}}\leq \frac{a+b+c}{2abc}$

Nhân $abc$ cho cả 2 vế thu được BĐT quen thuộc:

$\sum \sqrt{ab}\leq \sum a$




#569697 [CHUYÊN ĐỀ] CHỨNG MINH BẤT ĐẲNG THỨC

Đã gửi bởi Hoang Nhat Tuan on 03-07-2015 - 19:01 trong Bất đẳng thức và cực trị

Câu 10: Ta chứng minh bằng cách làm trội mỗi phân số của A bằng cách sử dụng BĐT

$\frac{n}{n+1}<\frac{n+1}{n+2}$

Khi đó $A<\frac{2}{3}.\frac{4}{5}...\frac{2n}{2n+1}$

Do đó: $A^2<(\frac{1}{2}.\frac{3}{4}.\frac{5}{6}...\frac{2n-1}{2n}).(\frac{2}{3}.\frac{4}{5}...\frac{2n}{2n+1})=\frac{1}{2n+1}$

=> ĐPCM




#572301 [CHUYÊN ĐỀ] CHỨNG MINH BẤT ĐẲNG THỨC

Đã gửi bởi Hoang Nhat Tuan on 14-07-2015 - 09:16 trong Bất đẳng thức và cực trị

$(23)\frac{x}{y}=a;...\Rightarrow abc=1.Prove:a^2+b^2+c^2\geqslant a+b+c;have:\sum a^2+2abc+1\geqslant 2\sum a\Leftrightarrow \sum a^2\geqslant \sum a+\sum a-3\geqslant \sum a$

Bài này có cách khác:$3(\frac{x^2}{y^2}+\frac{y^2}{z^2}+\frac{z^2}{x^2})\geq (\frac{x}{y}+\frac{y}{z}+\frac{z}{x})^2\geq 3(\frac{x}{y}+\frac{y}{z}+\frac{z}{x})$

=> ĐPCM




#572306 [CHUYÊN ĐỀ] CHỨNG MINH BẤT ĐẲNG THỨC

Đã gửi bởi Hoang Nhat Tuan on 14-07-2015 - 09:21 trong Bất đẳng thức và cực trị

Bài 20 sẽ thử lần lượt:

Với n=0 hoặc n=1 dễ thấy BĐT sai :D

Với n=2 thì BĐT trở thành: $2a^2b^2+2b^2c^2+2c^2a^2\leq a^4+b^4+c^4$

Dễ thấy với $a^2,b^2,c^2$ là độ dài 3 cạnh tam giác thì BĐT sai

Ta sẽ chứng minh n=3

Thật vậy: BĐT tương đương:

$3(a^4+b^4+c^4)\geq (a^2+b^2+c^2)^2$

$<=>a^4+b^4+c^4\geq a^2b^2+b^2c^2+c^2a^2$

P/s: Chú ý giả thiết n là số tự nhiên nhỏ nhất




#614520 [Hình học] THPT tháng 12: Chứng minh $PA=PL$.

Đã gửi bởi Hoang Nhat Tuan on 12-02-2016 - 17:44 trong Thảo luận đề thi VMEO IV

Untitled17.png

Gọi $D$ là giao của $AK$ và $EF$, $N$ là giao của $AM$ với $(O)$, $Q$ là giao điểm của $PF$ với $AB$ và $T$ là giao điểm của $PE$ với $AC$.

Trước tiên, ta sẽ chứng minh $D$ là giao của $AM$ và $EF$.

Giả sử $D'$ là giao của $AM$ và $EF$ vậy thì ta sẽ chứng minh $AD'\perp EF$:

Thật vậy: $TQ$ là đường trung bình của tam giác $PEF$ nên $TQ//EF$.

Nhận thấy tứ giác $AQPT$ nội tiếp (vì có $\widehat{AQP}=\widehat{ATP}=90^{\circ}$)

Nên $\widehat{APQ}=\widehat{ATQ}$

Kết hợp với $AP$ là đường đối trung của tam giác ABC dẫn đến:

$\widehat{MAT}+\widehat{ATQ}=\widehat{QAP}+\widehat{APQ}=90^{\circ}$

Do đó: $AD'\perp EF$ nên $D'\equiv D$

Ta có: $\Delta BNC=\Delta CPB(g-c-g)$ nên dẫn đến $BCNP$ là hình thang cân suy ra $OM$ là đường trung trực của $NP$ nên $MN=MP$

Nếu lấy điểm $D''$ thuộc tia $MA$ thỏa mãn $MD''=MN$ thì $D''$ sẽ đối xứng với $P$ qua $BC$.

Theo tính chất đường thẳng Steiner thì $F,D'',E$ thẳng hàng, mà $D''$ là giao của $EF$ với $AM$ nên $D''\equiv D$

$=>PD\perp AL$     (1)

Lại xét tam giác vuông $LAK$ có $DA=DK$ nên $DA=DL$ dẫn đến $D$ thuộc đường trung trực của $AL$  (2)

Từ (1) và (2) suy ra $PD$ chính là đường trung trực của $AL$.

Vậy $PA=PL$




#565797 [TOPIC] NHỮNG BÀI TOÁN CHƯA CÓ LỜI GIẢI

Đã gửi bởi Hoang Nhat Tuan on 14-06-2015 - 23:15 trong Đại số

Câu 15: $a\sqrt{b-1}+b\sqrt{a-1}\leq a.\frac{b-1+1}{2}+b.\frac{a-1+1}{2}=ab\leq abc$




#565795 [TOPIC] NHỮNG BÀI TOÁN CHƯA CÓ LỜI GIẢI

Đã gửi bởi Hoang Nhat Tuan on 14-06-2015 - 23:09 trong Đại số

Câu 25 giải rồi mà  :( http://diendantoanho...ãn-5x28y220412/




#614582 [Đại số]THPT tháng 12: $a^2+b^2+c^2-ab-bc-ca \geqslant k\left|...

Đã gửi bởi Hoang Nhat Tuan on 12-02-2016 - 21:10 trong Thảo luận đề thi VMEO IV

Một bài tương tự:

Tìm tất cả các số thực $k$ để bất đẳng thức sau đúng với mọi số thực không âm $a,b,c$:

\[k\left(\dfrac{a}{a+b}+\dfrac{b}{b+c}+\dfrac{c}{c+a}\right)\leqslant \dfrac{a^2+b^2+c^2}{ab+bc+ca}+\dfrac{3k-2}{2}\]

(Những viên kim cương trong bất đẳng thức toán học, trang 604)

Không biết hai bất đẳng thức này có tương đương không mà điểm rơi giống nhau và thằng $k$ cũng chả khác nhau mấy.

BĐT này tương đương với:

$2(a^2+b^2+c^2-ab-bc-ca)\geq\frac{k(a-b)(a-c)(b-c)(ab+bc+ca)}{(a+b)(b+c)(c+a)}$ nên nó gần như giống với đề tháng 12 này, chỉ khác ở chỗ một bên là $k$ với dấu giá trị tuyệt đối, một bên là $\frac{k}{2}$.